Difference between revisions of "2011 AMC 12A Problems/Problem 19"

(Created page with '== Problem == == Solution == == See also == {{AMC12 box|year=2011|num-b=18|num-a=20|ab=A}}')
 
(Problem)
Line 1: Line 1:
 
== Problem ==
 
== Problem ==
 +
At a competition with <math>N</math> players, the number of players given elite status is equal to <math>2^{1+\lfloor \log_{2} (N-1) \rfloor}-N</math>. Suppose that <math>19</math> players are given elite status. What is the sum of the two smallest possible values of <math>N</math>?
 +
 +
<math>
 +
\textbf{(A)}\ 38 \qquad
 +
\textbf{(B)}\ 90 \qquad
 +
\textbf{(C)}\ 154 \qquad
 +
\textbf{(D)}\ 406 \qquad
 +
\textbf{(E)}\ 1024 </math>
 +
 
== Solution ==
 
== Solution ==
 
== See also ==
 
== See also ==
 
{{AMC12 box|year=2011|num-b=18|num-a=20|ab=A}}
 
{{AMC12 box|year=2011|num-b=18|num-a=20|ab=A}}

Revision as of 02:36, 10 February 2011

Problem

At a competition with $N$ players, the number of players given elite status is equal to $2^{1+\lfloor \log_{2} (N-1) \rfloor}-N$. Suppose that $19$ players are given elite status. What is the sum of the two smallest possible values of $N$?

$\textbf{(A)}\ 38 \qquad \textbf{(B)}\ 90 \qquad \textbf{(C)}\ 154 \qquad \textbf{(D)}\ 406 \qquad \textbf{(E)}\ 1024$

Solution

See also

2011 AMC 12A (ProblemsAnswer KeyResources)
Preceded by
Problem 18
Followed by
Problem 20
1 2 3 4 5 6 7 8 9 10 11 12 13 14 15 16 17 18 19 20 21 22 23 24 25
All AMC 12 Problems and Solutions